+1 Daumen
693 Aufrufe

$$ \sum _{ n=1 }^{ \infty  }{ \frac { 1 }{ (3n-2)(3n+1) }  } $$

ich bin dann auf $$ \sum _{ n=1 }^{ \infty  }{ \frac { 1 }{ 3 } (\frac { 1 }{ (3n-2) }  } -\frac { 1 }{ (3n+1) } )$$

gekommen und habe mir die ersten Teile aufgeschrieben also:

$$ \frac { 1 }{ 3 } (\frac { 1 }{ 1 } -\frac { 1 }{ 4 } +\frac { 1 }{ 4 } -\frac { 1 }{ 7 } +\frac { 1 }{ 7 } ...)$$

da sich die Terme aufheben dachte ich $$ \frac { 1 }{ 3 } \ast \frac { 1 }{ 1 } =\frac { 1 }{ 3 } $$ ist richtig aber die letzte negative Stelle kann sich ja eigentlich nicht aufheben.

Wie komm ich da dann auf den Grenzwert?

Dankschön

Avatar von

1 Antwort

0 Daumen
 
Beste Antwort

" Dachte 1/3 ist richtig aber die letzte negative Stelle kann sich ja eigentlich nicht aufheben.

Wie komm ich da dann auf den Grenzwert? "

Im Grenzwert ist 1/3 exakt richtig. Grund: Die Folge (1 /(3n+1))_(nElementN) ist eine Nullfolge.

Also du hast erst mal 1/3( 1 + 1/(3k+3)) , wenn du von n=0 bis k summierst (k Element N grösser als sagen wir 2)

Nun machst du limes_(k gegen unendlich) 1/3( 1 + 1/(3k+3))  = 1/3( 1 + 0)  = 1/3 .

EDIT: So was nennt man übrigens Teleskopsumme. Ich habe oben diesen Tag ergänzt. 

Avatar von 162 k 🚀

Vielen Dank für die schnelle Antwort :)

Ein anderes Problem?

Stell deine Frage

Willkommen bei der Mathelounge! Stell deine Frage einfach und kostenlos

x
Made by a lovely community